Question

6. When a firm uses K units of capital and L units of labor, it can produce 2 units of output with the production function 2=

0 0
Add a comment Improve this question Transcribed image text
Answer #1

Cost function:

C = w*L + r*K

Given w=1 and r=2; the total cost is given as

C = L + 2K

a) K is fixed at K=16. Put the value in the production function, we get

Q = L0.5 + (16)0.5

Q = L0.5 + 4

For Q < = 4, the labor requirement is

L0.5 < = 4 - 4 = 0

L0.5 < = 0

It means, L < = 0

The short run cost will be, the cost of capital as labor can never be less than zero.

C = 0+12 * 16 = 32

----

b) Q > 4,

L0.5 > 4 - 4 = 0

L0.5 > 0

L > 0

The short run cost will be, the cost of capital and labor depending on the amount of output.

::C = (Q-4)2 +2 x 16 = Q? - 8Q + 48..................[ for Q>4].

Add a comment
Know the answer?
Add Answer to:
6. When a firm uses K units of capital and L units of labor, it can...
Your Answer:

Post as a guest

Your Name:

What's your source?

Earn Coins

Coins can be redeemed for fabulous gifts.

Not the answer you're looking for? Ask your own homework help question. Our experts will answer your question WITHIN MINUTES for Free.
Similar Homework Help Questions
  • A firm discovers that when it uses K units of capital and L units of labor,...

    A firm discovers that when it uses K units of capital and L units of labor, it is able to produce X = L^1/4*K^3/4 units of output. a. Draw the graph of isoquants in labor-capital plane. b. Suppose that the firm produces 24 units of output using 16 units of capital and 81 units of labor. Compute MRTS subscript LK. Compute the MPL. Compute the MPK. c. On the basis of your answer to part (b), is the equation MRTS...

  • A firm uses two types of inputs, labor (L) and capital (K), to produce an output,...

    A firm uses two types of inputs, labor (L) and capital (K), to produce an output, which is sold in a perfectly competitive market. The production function is given by y = f(L, K) = L 1 6 K 1 6 for all L, K ≥ 0. The price of labor is w > 0 and the price of capital is 1. Each unit of the output is sold at price p > 0. First, we consider the short-run decision...

  • A firm uses labor (L) and capital (K) as inputs, and has a short run cost function C=15+ 10q+ q2. Capital is fixed at K̅

    A firm uses labor (L) and capital (K) as inputs, and has a short run cost function C=15+ 10q+ q2. Capital is fixed at K̅ a. Give the formula for the firm's marginal cost function. Any method of deriving the marginal cost function is acceptable. (Hint: When calculating MC, you can assume that increases by a very, very small amount, so that q2 = q1 + ε ≈ q and q1 + q2 ≈ 2q.) b. Give the formula for the firm's...

  • Suppose a firm uses only two inputs, labor (L) and capital (K).

    Suppose a firm uses only two inputs, labor (L) and capital (K). In the short run, the amount of capital is fixed. If the price of labor is $4, and the marginal cost of production is $8, then the marginal product of labor is 4 2 32 12 None of the other answers are correct.

  • A firm produces output Q by using capital K and labor L in fixed proportions, i.e....

    A firm produces output Q by using capital K and labor L in fixed proportions, i.e. Q = F (K ,L ) = min {K, L/3}. The price of a unit of labor is w = 6, the price of a unit of capital is r = 2 and the price of output is p = 20. a) Draw the isoquant for Q = 8. b) Find the marginal product of labor. Suppose that (in part c and d) the...

  • The production of Florida strawberries uses two inputs: labor (L) and capital (K). The following production...

    The production of Florida strawberries uses two inputs: labor (L) and capital (K). The following production function describes how these inputs are combined to produce bushels of oranges. f(L,K) = 5(1/2 + 3K1/2 1) Determine what kind of returns to scale this production function exhibits (HINT: labor is the "x" variable - the one that goes on the horizontal axis). 2) What is the formula for that kind of returns to scale? (HINT: use f(L,K)) 3) What is the general...

  • 2. (12 total points) Suppose a firm can use either Capital (K) or Labor (L) in...

    2. (12 total points) Suppose a firm can use either Capital (K) or Labor (L) in a production process. The firm's production function is given by Q = 5L + 15K. The price of Capital is $20 per unit and the price of Labor is $8 per unit. a) (4 points) What is the firm's Total Cost function? TC(Q)= b) (8 points) Suppose the firm is producing 30 units of output (Q = 30). Using a graph, draw the firm's...

  • Consider a production function of three inputs, labor, capital, and materials, given by Q= LKM

     Consider a production function of three inputs, labor, capital, and materials, given by Q= LKM. The marginal products associated with this production function are as follows: MPL = KM, MPk = LM, and MPM = LK. Let w = 5, r = 1, and m = 2, where m is the price per unit of materials. (a) Suppose that the firm is required to produce Q units of output. Show how the cost-minimizing quantity of labor depends on the quantity Q....

  • NEED ALL ANSWERS PLEASE Problem 3 [24 marks] A competitive firm uses two inputs, capital (k) and labour (), to produce...

    NEED ALL ANSWERS PLEASE Problem 3 [24 marks] A competitive firm uses two inputs, capital (k) and labour (), to produce one output, (y). The price of capital, W, is S1 per unit and the price of labor, wi, is SI per unit. The firm operates in competitive markets for outputs and inputs, so takes the prices as given. The production function is f(k,l) 3k025/025. The maximum amount of output produced for a givern amount of inputs is y(k, l)...

  • 1. The production function of a firm is f(1,k) = Vlk where l is labor and...

    1. The production function of a firm is f(1,k) = Vlk where l is labor and k is capital/machinery. a. In the short run, if the quantity of capital is fixed at 64, derive the short run total cost SC(q), average cost SAC(q), and marginal cost SMC(q) of this firm. Assume each input costs $1 per unit. At what output does the minimum of SAC(q) occur? b. If labor and capital cost r and w respectively, and the quantity of...

ADVERTISEMENT
Free Homework Help App
Download From Google Play
Scan Your Homework
to Get Instant Free Answers
Need Online Homework Help?
Ask a Question
Get Answers For Free
Most questions answered within 3 hours.
ADVERTISEMENT
ADVERTISEMENT
ADVERTISEMENT